Limit of Integral: x $\rightarrow$ 0, $\int{\sin{t^2}}dt$ 0 to 3

In summary, the problem is asking for the limit as x approaches 0 of a fraction with a constant in the numerator and x^3 in the denominator, where the integral is from 0 to 3 of sin(t^2). The solution may be simplified if the upper limit of the integral is changed to x and the divisor is x^3.
  • #1
ultrazyn
2
0
limit as x approaches 0 of [(1/x^3) * integral from 0 to 3 of (sin(t^2))dt]

is the way to solve this using mean value theorem for integrals?

[tex]\lim_{x\rightarrow0}\frac{1}{x^3}\int{\sin{t^2}}dt[/tex] the integral is from 0 to 3
not sure if i did the latex right
 
Last edited:
Physics news on Phys.org
  • #2
limit as x approaches 0 of [(1/x^3) * integral from 0 to 3 of (sin(t^2))dt]
[tex]\lim_{x \rightarrow 0} \frac{1}{x^3} \int_0^3 \sin{t^2} dt[/tex]
 
Last edited:
  • #3
If the integral is, in fact, from 0 to 3, then it is just a constant. This is asking for
[tex]\lim_{x \rightarrow 0} \frac{c}{x^2}[/tex]
where c is a non-zero constant. Looks pretty easy to me.
 
  • #4
The problem makes more sense (as a problem) if the upper limit of the integral is x, not 3. In that case, the answer is 1/3, assuming that the divisor is x3, not x2.
 

1. What is the limit of the integral as x approaches 0?

The limit of the integral as x approaches 0 is 0. This means that the value of the integral approaches 0 as x gets closer and closer to 0.

2. How is the limit of the integral calculated?

The limit of the integral is calculated by taking the integral of the function and then evaluating it at the limit point. In this case, the integral of sin(t^2) is taken and then evaluated at x=0.

3. Why is the limit of the integral important?

The limit of the integral is important because it helps us understand the behavior of a function as it approaches a certain point. It also allows us to determine if a function is continuous at a specific point.

4. What is the significance of the interval 0 to 3 in the integral?

The interval 0 to 3 represents the range of values for which the integral is being evaluated. In this case, the integral is being evaluated from 0 to 3, meaning that the function is being integrated over the interval from 0 to 3.

5. Is it possible to change the limits of integration in this integral?

Yes, it is possible to change the limits of integration in this integral. The value of the integral may change depending on the new limits of integration, but the limit of the integral will remain the same as long as the new limits are within the interval of 0 to 3.

Similar threads

Replies
2
Views
293
  • Calculus
Replies
3
Views
2K
Replies
3
Views
1K
Replies
4
Views
353
Replies
12
Views
1K
  • Calculus
Replies
6
Views
1K
  • Calculus
Replies
4
Views
1K
Replies
3
Views
1K
Replies
16
Views
1K
Replies
3
Views
334
Back
Top